4
$\begingroup$

I came across this complex function in my work $f(z)=\frac{e^z-1}{z}$. Is there a reference to $f(z)$? What is its name in the literature? More importantly, is the function inversible? If so, what is $f^{-1}(z)$? Thanks.

$\endgroup$
6
  • $\begingroup$ I don't know about the name, but certainly it can't be globally invertible. For example, it assumes the value 0 infinitely often. The only invertible global holomorphic functions are the polynomials of degree 1. $\endgroup$
    – Angelo
    May 2, 2012 at 10:08
  • 4
    $\begingroup$ en.wikipedia.org/wiki/Bernoulli_number#Generating_function is relevant. $\endgroup$ May 2, 2012 at 10:31
  • $\begingroup$ Thanks, if I write the function as a series instead, i.e. $f(z) = \sum_{k=0}^\infty \frac{z^k}{(k+1)!}$, is it invertible? $\endgroup$ May 2, 2012 at 11:19
  • $\begingroup$ It is strictly increasing on the reals, so it is invertible there. $\endgroup$ May 2, 2012 at 12:51
  • 3
    $\begingroup$ The Wikipedia article gives a series for 1/f(z), but the question was about $f^{-1}(z)$. $\endgroup$ May 2, 2012 at 15:00

4 Answers 4

12
$\begingroup$

Let $y=(e^z-1)/z$ and $x=-1/y$. Then $xe^x=(x-z)e^{x-z}$. Hence $$x-z=W(xe^x).$$ Here W is an appropriately chosen branch of the Lambert function (ProductLog[-1,.] in Mathematica).

$\endgroup$
4
  • $\begingroup$ Interesting. Of course $x=W(x e^x)$ for some other branch of $W$. $\endgroup$ May 2, 2012 at 15:25
  • 1
    $\begingroup$ ...so I guess this means: solutions to $y=f(z)$ are $$ z=-\frac{1 + W_k \left(-\frac{\operatorname{e} ^{-1/y}}{y}\right) y}{y} $$ where $W_k$ are the branches of the Lambert W function. $\endgroup$ May 2, 2012 at 15:37
  • $\begingroup$ We need to rule out the principal branch of the Lambert function, which would simply give z=0. $\endgroup$ May 2, 2012 at 15:51
  • $\begingroup$ Thanks. This is precisely the answer I have been looking for. $\endgroup$ May 2, 2012 at 16:32
7
$\begingroup$

As for the name, according to wikipedia the Todd genus is given by:

$$\mathrm{Td}(z)=\frac{z}{1-e^{-z}}.$$

So, $f(z)=1/\mathrm{Td}(-z)$.

$\endgroup$
4
  • 3
    $\begingroup$ Interesting, but does not answer the question. Why the upvotes??? $\endgroup$ May 2, 2012 at 19:55
  • 4
    $\begingroup$ @András Bátkai: probably because it's a near-answer ["$1/{\rm Td}(-z)$" feels closer to a named function than "$(e^z-1)/z$"] and makes a possibly unexpected connection with research-level mathematics. $\endgroup$ May 3, 2012 at 0:56
  • 1
    $\begingroup$ @András Bátkai: Let's not be competitive - if someone posts a useful answer/remark I'm glad to upvote it. $\endgroup$
    – Qfwfq
    May 3, 2012 at 7:50
  • 3
    $\begingroup$ There was no competitiveness. I was just surprised that the accepted answer got (at the time) less upvote then this one. Which is, of course, informative. $\endgroup$ May 3, 2012 at 8:20
3
$\begingroup$

the coefficients of $f^{-1}(z)$ is just bernoulli numbers

$$\frac{z}{e^z-1}=\sum_{n=0}^{\infty}B_n\frac{z^n}{n!}$$

and we have Bernoulli numbers of higher order $\alpha$

$$\big(\frac{z}{e^z-1}\big)^\alpha=\sum_{n=0}^{\infty}B_n^{(\alpha)}\frac{z^n}{n!}$$

where $\alpha=-1$ in the case $f(z)$ and Todd genus can be computed in terms of Bernoulli numbers

$\endgroup$
0
$\begingroup$

A related thread (about relations between two real branches of lambert function):

Equation between the two branches of the lambert w function

A (perhaps more complete?) solution involving both the real branches could be written as:

$z=W_0(-\frac{e^\frac{-1}{f}}{f})-W_{-1}(-\frac{e^\frac{-1}{f}}{f})=-\frac{1}{f}-W_{-1}(-\frac{e^\frac{-1}{f}}{f})$ for $f >1 $

$z=W_{-1}(-\frac{e^\frac{-1}{f}}{f})-W_{0}(-\frac{e^\frac{-1}{f}}{f})=-\frac{1}{f}-W_{0}(-\frac{e^\frac{-1}{f}}{f})$ for $f < 1 $

$\endgroup$
2
  • 1
    $\begingroup$ This does not provide an answer to the question. To critique or request clarification from an author, leave a comment below their post - you can always comment on your own posts, and once you have sufficient reputation you will be able to comment on any post. $\endgroup$
    – Stefan Kohl
    Feb 12, 2015 at 21:47
  • $\begingroup$ I have edited my post adding details and providing an answer to the question. $\endgroup$ Feb 14, 2015 at 9:29

Your Answer

By clicking “Post Your Answer”, you agree to our terms of service and acknowledge you have read our privacy policy.

Not the answer you're looking for? Browse other questions tagged or ask your own question.